9 is subtracted from 3 times the sum of 4 and 2.​

Answers

Answer 1

Answer:

9

Step-by-step explanation:

This problem is represented below.

=3(4+2)-9

=3(6)-9

=18-9

=9

Answer 2
3 x (4 + 2) - 9 is the representative equation for this sentence. According to pemdas, we first evaluate parenthesis. 4 + 2 = 6. In the next order of pemdas, we multiply. 3 x 6 = 18. Lastly, we can subtract the final two numbers to get our final result for the sentence. 18 - 9 = 9. Therefore, our final answer to this equation is 9.

Your final answer: 3 x (4 + 2) - 9 = 9.

Related Questions

Need help with the following question

Answers

3 times. Draw a line 1 unit up, and you can see that there are 3 intersections.

To purchase 13700 worth of restaurant equipment for her business Maria made a down payment of 1500 and took out a business loan for the rest after 3 years of paying monthly payments of 371.16 she finally paid off the loan
What was the total amount Maria ended up paying for the equipment

How much internet did Maria pay on the loan

Answers

The total amount Maria ended up paying for the equipment will be $14,861.76. And The interest of Maria on the loan will be 8.48%.

What is Algebra?

The analysis of mathematical representations is algebra, and the handling of those symbols is logic.

To purchase 13700 worth of restaurant equipment for her business.

Maria made a down payment of 1500 and took out a business loan for the rest, after 3 years of paying monthly payments of 371.16 she finally paid off the loan.

The total amount Maria ended up paying for the equipment will be

Total amount = 371.16 × 3 × 12 + 1500

Total amount = $14,861.76

The interest of Maria on the loan will be

Interset = [(14861.76 – 13700) / 13700] x 100

Interset = 8.48%

More about the Algebra link is given below.

https://brainly.com/question/953809

#SPJ1

(7 + 7i)(2 − 2i)
(a) Write the trigonometric forms of the complex numbers. (Let
0 ≤ < 2.)

(7 + 7i) =

(2 − 2i) =



(b) Perform the indicated operation using the trigonometric forms. (Let
0 ≤ < 2.)



(c) Perform the indicated operation using the standard forms, and check your result with that of part (b).

Answers

The complex number  -7i into trigonometric form is 7 (cos (90) + sin (90) i) and  3 + 3i in trigonometric form is 4.2426 (cos (45) + sin (45) i)

What is a complex number?

It is defined as the number which can be written as x+iy where x is the real number or real part of the complex number and y is the imaginary part of the complex number and i is the iota which is nothing but a square root of -1.

We have a complex number shown in the picture:

-7i(3 + 3i)

= -7i

In trigonometric form:

z = 7 (cos (90) + sin (90) i)

= 3 + 3i

z = 4.2426 (cos (45) + sin (45) i)

[tex]\rm 7\:\left(cos\:\left(90\right)\:+\:sin\:\left(90\right)\:i\right)4.2426\:\left(cos\:\left(45\right)\:+\:sin\:\left(45\right)\:i\right)[/tex]

[tex]\rm =7\left(\cos \left(\dfrac{\pi }{2}\right)+\sin \left(\dfrac{\pi }{2}\right)i\right)\cdot \:4.2426\left(\cos \left(\dfrac{\pi }{4}\right)+\sin \left(\dfrac{\pi }{4}\right)i\right)[/tex]

[tex]\rm 7\cdot \dfrac{21213}{5000}e^{i\dfrac{\pi }{2}}e^{i\dfrac{\pi }{4}}[/tex]

[tex]\rm =\dfrac{148491\left(-1\right)^{\dfrac{3}{4}}}{5000}[/tex]

=21-21i

After converting into the exponential form:

[tex]\rm =\dfrac{148491\left(-1\right)^{\dfrac{3}{4}}}{5000}[/tex]

From part (b) and part (c) both results are the same.

Thus, the complex number  -7i into trigonometric form is 7 (cos (90) + sin (90) i) and  3 + 3i in trigonometric form is 4.2426 (cos (45) + sin (45) i)

Learn more about the complex number here:

brainly.com/question/10251853

#SPJ1

!!!!!!!!!!!!!!!!!!!!!!!!!!!!!!!!!!!!!!!!

Answers

[tex]\quad \huge \quad \quad \boxed{ \tt \:Answer }[/tex]

[tex] \texttt{ \:The absolute maxima of f is f(-8) = 6} [/tex]

____________________________________

[tex] \large \tt Solution \: : [/tex]

Absolute maxima is the maximum possible value for a given x, of a function.

and here, the maximum value is at -8, and the maximum value is 6.

[tex]\qquad \tt \rightarrow \: maximum - \: \: f( - 8) = 6[/tex]

Answered by : ❝ AǫᴜᴀWɪᴢ ❞

What are the solutions to the equations y= 4x^2+5x-6

Answers

Answer:

D. x = -2; x = 3/4

Step-by-step explanation:

Read the values of a, b, and c from the quadratic equation: a is the number in front of x^2, b is the number in front of x, c is the number at the end. In our case: a = 4,b = 5,c = −6 The formula for the roots is =  [tex]\frac{-b +-\sqrt{b^{2}-4ac } }{2a\\}[/tex]

A sells an article to B at profit of 20% and B sells it to C at a profit of 25%. If C buys it for Rs.225, what did A pay for it?​

Answers

Answer:

A pay 120 for it

Let the cost price be X

A Sells to B

profit = 20%

= cost price + 20%

= X + 20/100 of X

= 6x/ 5

B sells to C

profit = 25%

= Cost price + + 25%

= cost price + 25%

= 6x/5 + 25/100*6x/5

= 6x/5.+ 3x/10.

= 3x/2

3x/2 = 225.

3x = 450

x = 150.

sp = C. p + profit

S. p - profit = C. p

X - X/5

4x/5

4(150)/5.

120

I need help ASAP please

Which statement is true about whether Z and B are independent events?

A: Z and B are independent events because P(Z | B) =P(Z).
B: Z and B are independent events because P(Z | B) = P(B).
C: Z and B are not independent events because P(Z | B) ≠ P(Z).
D: Z and B are not independent events because P(Z | B) ≠ P(B).

Answers

A

Explanation:

Independent event means P(Z|B)=P(Z)

P(Z|B)=(Z ∩ B)/B
=(126)/280
=.45

P(Z)=297/660
=.45

Therefore they are independent events, and the answer is A.
The answer should be Option A

What are m and b in the linear equation y=16+6x

Answers

Answer: B=16 and m=6

Step-by-step explanation:

in y=mx+b the number next to x is always the m/slope and the number without a variable is always the b.

Question 10 of 10
Rewrite the following linear equation in slope-intercept form. Write your
answer with no spaces.
v+2=4(x-3)
Answer here

Answers

y+2=4(x-3)
y+2=4x-12
y+2-2=4x-12-2
y=4x-14

can someone help me asap :)

Answers

Answer:

Option 2 - TU is perpendicular to MN and MN is parallel to PQ

angle M = angle P ( 90°)

so MN is parallel to pq

MN makes 90° with UT so it is perpendicular

A number is equal to the sum of half a second number and 3. The first number is also equal to the sum of one- quarter of the second number and 5. The situation can be represented by using the graph below, where x represents the second number. 16 NO 8 K 3₂ 5 2 4 st 6 8 10 12 14 16 Which equations represent the situation? Hurry ​

Answers

5 1/2
this is how we do it, jus learn abt it

One positive number is 8 times another number. Their difference is 70.
Which of the following equations could be used to find the numbers?

Answers

Answer:

X equals 10.

8x - x = 70

Step-by-step explanation:

8 x 10 = 80

80 - 10 = 70

Find the area of the trapezoid. TOP 11ft, RIGHT4√3ft , BOTTOM 15ft ,LEFT 8ft

Answers

Answer:

[tex]52\sqrt{3} ft^{2}[/tex]

Step-by-step explanation:

Please refer to the attached picture.

First we will find the area of rectangle BCDE.

Area of Rectangle = Length x Breadth = DE x CD

= 11 x [tex]4\sqrt{3}[/tex]

[tex]=44\sqrt{3} ft^{2}[/tex]

Next we will find Area of Triangle ABE.

Area of Triangle = 0.5 x Base x Height

[tex]0.5*4*4\sqrt{3} \\=8\sqrt{3} ft^{2}[/tex]

Area of Trapezoid = Area of Rectangle + Area of Triangle

[tex]=44\sqrt{3} +8\sqrt{3} \\=52\sqrt{3} ft^{2}[/tex]

Answer:

A = 52[tex]\sqrt{3}[/tex] ft² ≈ 90.1 ft²

Step-by-step explanation:

the area (A) of a trapezoid is calculated as

A = [tex]\frac{1}{2}[/tex] h (b₁ + b₂ )

where h is the perpendicular height and b₁ , b₂ the parallel bases

here h = 4[tex]\sqrt{3}[/tex] , b₁ = 15 , b₂ = 11 , then

A = [tex]\frac{1}{2}[/tex] × 4[tex]\sqrt{3}[/tex] × (15 + 11)

   = 2[tex]\sqrt{3}[/tex] × 26

   = 52[tex]\sqrt{3}[/tex] ft²

   ≈ 90.1 ft² ( to the nearest tenth )

March 8, 2017, one U.S. dollar was worth 66.79 Indian rupees.
a) On that date, how many dollars was 110.66 rupees worth?
Round your answer to the nearest hundredth of a dollar. I need help with this question.

Answers

[tex] \huge \tt \underline {\green{Answer}}[/tex]

If on March 8, 2017 , one U.S. dollar worth 66.79 Indian rupees

ie. $1 = Rs 66.79

$ 1 = 66.79 × 1

$ ? = 110.66

$ = New / old

$ = 110.66 / 66.79

$ = 1.65683485552

or

$1.66 = 110.66

four big water bottles can hold 8 gallaons how much can 10 big water gallons hold

Answers

4 can hold 8 gallons = 8/4 = 2 big bottle per gallons.

For 10 big bottles = 10 x 2 = 20 gallons.

If you are looking for how many bottles can be filled with 10 gallons, the answer is 5.
Let x represent the gallons you are finding
If 4=8 then,
10=x
You cross multiply
4x = 10*8
4x /4= 80/4
x = 20

which expression is equivalent to...​

Answers

Answer:

B

Step-by-step explanation:

Which of the following terms best fits this definition?

The angle between two sides of a triangle.

Select one:

AAS Theorem


SAS Postulate


Included Side


ASA Postulate


HL Congruence Theorem


Included Angle


SSS Postulate

Answers

Step-by-step explanation:

hope you can understand

How many pounds of candy that sells for ​$0.82 per lb must be mixed with candy that sells for ​$1.36 per lb to obtain 9 lb of a mixture that should sell for ​$0.91 per​ lb?

Answers

7.5 pounds of the $0.82 per lb candy must be used in the mixture.

How many pounds of each candy should we use?

First, let's define the variables:

x = pounds of the $0.82 candy used.y = pounds of the $1.36 candy used.

We want to make 9 lb of mixture, then:

x + y = 9.

And the price of these 9 pounds must be $0.91, then we can write:

x*$0.82 + y*$1.36 = 9*$0.91 = $8.19

Then we have a system of equations:

x + y = 9.

x*$0.82 + y*$1.36 = $8.19

We can isolate y on the first equation so we get:

y = 9 - x

Now we can replace that on the other equation:

x*$0.82 + (9 - x)*$1.36 = $8.19

And now we can solve this for x.

x*($0.82 - $1.36) = $8.19 - 9*$1.36

-x*$0.54 = -$4.05

x = (4.05/0.54) = 7.5

So 7.5 pounds of the $0.82 per lb candy must be used in the mixture.

If you want to learn more about systems of equations:

https://brainly.com/question/13729904

#SPJ1

It has been estimated that only about 40% of California residents have adequate earthquake supplies suppose you randomly survey 22 California residents we are interested in the number who have adequate earth quake supplies

Answers

The answers are as follows:

a) X denotes the number of the california residents that have adequate earthquake insurance

b)  x = 1 ,2 ,3 ......

c) P( X=x ) = 0.3(1-0.3) ^ (x-1)

d)  P( X=1) +  P( X=2)  + P( X=3) +  P( X=4)

e) 0.49

f)0.42

g)0.33

What is probability?

It is a branch of mathematics that deals with the occurrence of a random event.

The complete question is

It has been estimated that only about 30% of California residents have adequate earthquake supplies. Suppose we are interested in the number of California residents we must survey until we find a resident who does not have adequate earthquake supplies. a. In words, define the random variable X. b. List the values that Xmay take on. c. Give the distribution of X.X~ _____(_____,_____) d. WhatistheprobabilitythatwemustsurveyjustoneortworesidentsuntilwefindaCaliforniaresidentwhodoes not have adequate earthquake supplies? e. What is the probability that we must survey at least three California residents until we find a California resident who does not have adequate earthquake supplies? f. HowmanyCaliforniaresidentsdoyouexpecttoneedtosurveyuntilyoufindaCaliforniaresidentwhodoesnot have adequate earthquake supplies? g. How many California residents do you expect to need to survey until you find a California resident who does have adequate earthquake supplies?

given that 30% of California residents have adequate earthquake supplies.

a) variable X denotes the number of the california residents that have adequate earthquake insurance

b) x = 1 ,2 ,3 ......

c) p=0.3

P( X=x ) = 0.3(1-0.3) ^ (x-1)

d) P( X=1) +  P( X=2)  + P( X=3) +  P( X=4)

= 0.3(1-0.3) ^ 0 + 0.3(1-0.3) ^1 + 0.3(1-0.3)^ 2 +...

e) P( X≥ 3) = 1- P(X<3)

= 1- (P(X=1) + (X=2))

= 1- 0.051

= 0.49.

f) E(X)= 1/P

p is the resident who does not have adequate earthquake supplies

p= 1-0.3

p = 0.7

E(X) = 1/0.7

= 0.42

g) E(X) =  1/q

= 1/0.3

= 3.333

Learn more about this concept here:

https://brainly.com/question/15512751

#SPJ1

What is the simplified form of √ 144x36?

Answers

Answer:

12x^18

Step-by-step explanation:

The Strikers soccer team has 20 members, and 8 of them play offense. What percent of the team members play offense?

Answers

Answer:

40%

Step-by-step explanation:

We already have our first value 20 and the second value 8. Let's assume the unknown value is Y which answer we will find out.

As we have all the required values we need, Now we can put them in a simple mathematical formula as below:

Step 1 ⇒ Y = 8/20

By multiplying both numerator and denominator by 100 we will get:

Step 2 ⇒ Y = 8/20 × 100/100 = 40/100

Step 3 ⇒ Y = 40

Finally, we have found the value of Y which is 40 and that is our answer.

find the slope of the line that passes through (3,10) and (1,17)​

Answers

Answer:

[tex]m=- \frac{7}{2}[/tex]

Step-by-step explanation:

The slope of a line passing through the two points [tex]\displaystyle{\large{{P}={\left({x}_{{1}},{y}_{{1}}\right)}}}[/tex] and[tex]\displaystyle{\large{{Q}={\left({x}_{{2}},{y}_{{2}}\right)}}}[/tex] is given by [tex]\displaystyle{\large{{m}=\frac{{{y}_{{2}}-{y}_{{1}}}}{{{x}_{{2}}-{x}_{{1}}}}}}[/tex].

We have that [tex]x_1=3[/tex], [tex]y_1=10[/tex], [tex]x_2=1[/tex], [tex]y_2=17[/tex].

Plug the given values into the formula for slope: [tex]m=\frac{\left(17\right)-\left(10\right)}{\left(1\right)-\left(3\right)}=\frac{7}{-2}=- \frac{7}{2}[/tex]

Answer: the slope of the line is [tex]m=- \frac{7}{2}[/tex].

Answer:

slope = - [tex]\frac{7}{2}[/tex]

Step-by-step explanation:

calculate the slope m using the slope formula

m = [tex]\frac{x_{2}-y_{1} }{x_{2}-x_{1} }[/tex]

with (x₁, y₁ ) = (3, 10 ) and (x₂, y₂ ) = (1, 17 )

m = [tex]\frac{17-10}{1-3}[/tex] = [tex]\frac{7}{-2}[/tex] = - [tex]\frac{7}{2}[/tex]

what is the ratio of the radius of circle a to the radius of circle b?

Answers

Answer:

3:1

Step-by-step explanation:

Assuming that the edges of the circle are supposed to line up with the dotted lines of the graph, all you have to do is count how many lines are between the middle of the circle and the edge (either directly vertical or horizontal due to the graph we are using). We can see that the radius of circle A is approximately 3 units, while the radius of circle B is approximately 1 unit. So the ratio of the radius of circle A to the radius of circle B is 3:1.

Answer:

3:1

Step-by-step explanation:

because once count the radius of the big circle that is 3 unit and the radius of small circle is 1 unit

1
2
3
5
10
Two runners are saving money to attend a marathon.
The first runner has $110 in savings, received a $45 gift
from a friend, and will save $25 each month. The
second runner has $50 in savings and will save $60
each month.
After how many months will both runners have the
same amount of money?
02
O 3

Answers

Answer:

3 months

Step-by-step explanation:

The first runner: $110 + $45 = $155 starting out, plus 25x for $25 each month.

The second runner: $50 starting out, plus 60x for $60 each month.

To find out when both runners have the same amount of money, we will set the expressions equal to each other and solve.

155 + 25x = 60x + 50

105 = 35x

x = 3

Brainliest, please :)

Select the correct answer from each drop-down menu.
Consider the function Ax) = 3x+ 1 and the graph of the function g(x) shown below.

Answers

Answer:

write three methods of writing set. Give one example of each methods.

|4x + 7| − 4 = 20
Can anyone help me with this

Answers

Answer:

x = -31/4  or x = 17/4

Step-by-step explanation:

|4x + 7| − 4 = 20

⇔ |4x + 7| − 4 + 4 = 20 + 4

⇔ |4x + 7| = 24

⇔ 4x + 7 = 24  or 4x + 7 = -24

⇔ 4x = 24 - 7 or 4x = -24 - 7

⇔ 4x = 17 or 4x = -31

⇔ x = 17/4 or x = -31/4

Answer:

x = [tex]\frac{17}{4}[/tex]    (17/4 = 4.25)

or x =  [tex]-\frac{31}{4}[/tex]  (-31/4 = -7.75)

Step-by-step explanation:

| | is notation for absolute value

absolute value - the distance that a number is from 0

> essentially, you can think of absolute value as the "positive version" of whatever is inside of the | |

if we have | x | = 20, we could really have (without the | | ) two versions of x

either | x | = 20  ; or | -x | = 20  {because a negative x inside of the | | has the same value as positive x}

we set this up as two equations:

x = 20        or         -x = 20

                                ^ {multiply by -1}

x = 20  or   x = -20

now, let's plug our understanding into the equation

|4x + 7| - 4 = 20

first, we should simplify our equation to:

|4x + 7| - 4 = 20

           + 4   + 4

 |4x + 7|   =   24

now, let's separate this absolute value equation into two separate equations:

4x + 7 = 24:

      - 7    - 7  {subtract 7 from both sides to isolate x}

4x = 17

÷4  ÷4        {divide both sides by 4 to get 1x}

x= [tex]\frac{17}{4}[/tex]

or,

4x + 7 = -24:

      - 7    -7   {subtract 7 from both sides to isolate x}

4x     =    -31

÷4           ÷4   {divide both sides by 4 to get 1x}

 x = [tex]-\frac{31}{4}[/tex]

so, we know that

x = [tex]\frac{17}{4}[/tex]    (17/4 = 4.25)

or x =  [tex]-\frac{31}{4}[/tex]  (-31/4 = -7.75)

hope this helps!!

Find an equation for the line that passes through the point P(-5,-3) and is parallel to the line
7x + 4y
10. Use exact values.

Answers

-------------------------------------------------------------------------------------------------------------

Answer:  [tex]\textsf{y = -1.75x - 11.75}[/tex]

-------------------------------------------------------------------------------------------------------------

Given:  [tex]\textsf{Goes through (-5, -3) and parallel to 7x + 4y = 10}[/tex]

Find:  [tex]\textsf{The equation in slope-intercept form}[/tex]

Solution: We need to first solve for y in the equation that was provided so we can determine the slope.  Then we plug in the values into the point-slope form, distribute, simplify, and solve for y to get our final equation.

Subtract 7x from both sides

[tex]\textsf{7x - 7x + 4y = 10 - 7x}[/tex][tex]\textsf{4y = 10 - 7x}[/tex]

Divide both sides by 4

[tex]\textsf{4y/4 = (10 - 7x)/4}[/tex][tex]\textsf{y = (10 - 7x)/4}[/tex][tex]\textsf{y = 10/4 - 7x/4}[/tex][tex]\textsf{y = 2.5 - 1.75x}[/tex]

Plug in the values

[tex]\textsf{y - y}_1\textsf{ = m(x - x}_1\textsf{)}[/tex][tex]\textsf{y - (-3) = -1.75(x - (-5))}[/tex]

Simplify and distribute

[tex]\textsf{y + 3 = -1.75(x + 5)}[/tex][tex]\textsf{y + 3 = (-1.75 * x) + (-1.75 * 5)}[/tex][tex]\textsf{y + 3 = -1.75x - 8.75}[/tex]

Subtract 3 from both sides

[tex]\textsf{y + 3 - 3 = -1.75x - 8.75 - 3}[/tex][tex]\textsf{y = -1.75x - 8.75 - 3}[/tex][tex]\textsf{y = -1.75x - 11.75}[/tex]

Therefore, the final equation in slope-intercept form that follows the information that was provided is y = -1.75x - 11.75

A woman passed gas silently. I said "it stinks", and she said "I apologize. Excuse me". Why did she say both of those things?

Answers

She says “I apologise” because she feels bad that she made a bad smell and she says “excuse me” because she wants you to accept her apology

Find the value below using your calculator. Round to the nearest hundredth. e³ Find the value below using your calculator . Round to the nearest hundredth . e³​

Answers

The value of e³ = 20.079.

What is Euler's number , e ?

e is an irrational number , It is a numerical constant used in mathematical constant .

The value of e = 2.718

It is asked in the question to determine the value of e³

Therefore

e³ = ( 2.718)³

e³ = 20.079

Therefore the value of e³ = 20.079.

The image of the calculation is attached with the answer.

To know more about Euler's number

https://brainly.com/question/573685

#SPJ1

A hyperbola centered at (7, 0) has a focus at (7, 5) and vertex at (7, 4). Which is the equation of the hyperbola in standard form?

quantity x minus 7 end quantity squared over 16 minus y squared over 9 equals 1
quantity x minus 7 end quantity squared over 25 minus y squared over 16 equals 1
y squared over 16 minus quantity x minus 7 end quantity squared over 9 equals 1
y squared over 25 minus quantity x minus 7 end quantity squared over 16 equals 1

Answers

Based on the calculations, the equation of this hyperbola in standard form is: A. [tex]\frac{x\;-\;7}{16} + \frac{y}{9} = 1[/tex].

How to determine the equation of a hyperbola?

Mathematically, the equation of a hyperbola in standard form is given by:

[tex]\frac{x\;-\;h}{a^2} + \frac{x\;-\;k}{b^2} = 1[/tex]

Given the following data:

Center (h, k) = (7, 0)

Vertex (h+a, k) = (7, 4)

Focus = (h+c, k) = (7, 5)

Also, we can deduce that the value of a and c are 4 and 5 respectively.

For the value of b, we would apply Pythagorean's theorem:

c² = a² + b²

b² = c² - a²

b² = 5² - 4²

b² = 9.

Substituting the parameters into the standard equation, we have:

[tex]\frac{x\;-\;7}{4^2} + \frac{y\;-\;0}{3^2} = 1\\\\\frac{x\;-\;7}{16} + \frac{y}{9} = 1[/tex]

Read more on hyperbola here: https://brainly.com/question/3405939

#SPJ1

Other Questions
Point P has coordinates P(-4, 3).What are the coordinates of the image P" after I translate point P: 1unit to the right 7 units down and then I reflect its image across they-axis?The x-coordinate of P" is:The y-coordinate of P" is: Determine whether it is a function A graph displays point A (5, 3, 4) and point B (4, 2, 6). Calculate the approximate distance each point is from the origin. Round to the nearest tenth. Which point is farther from the origin, and by how much? Estimate the solution to the system of equations.You can use the interactive graph below to find the solution.-3x+3y=9 2x-7y=-14Alse please don't take longChoices are attached Put the causes of the Great Dust Bowl in the correct order. What is the function of lacI in the regulation of the lac operon? It is the operator that mediates the transcription of the structural genes. It is a structural gene that encodes permease. It encodes a repressor that, in the absence of lactose, binds to the lac operator and blocks expression of the structural genes. It is the promoter that mediates the transcription of the operon. It encodes a repressor that, in the presence of lactose, binds to the lac operator and blocks expression of the structural genes. A store finds that the number of shirts sold increases each week. In the first week, only 15 shirts were sold. In the next week, 22 shirts were sold and in the third week 29 shirts were sold. The number of shirts sold each week represents an arithmetic sequence.What is the explicit rule for the arithmetic sequence that defines the number of shirts sold in week n? There are 250 students in a school and there are 120 more boys than girls. What percentageof the students are boys? f(x) = 2x2 + 3x + 5, i need this answer a.s.a.p!!! A _______ random variable has infinitely many values associated with measurements. PLS HELP ME WITH THIS PROBLEM 8. Mrs. Reyes has 4 mangoes, 5 guavas, and 2 bananas in her basket. What is the probability that one can pick a guava? A. 2 out of 11 or 2/11 B. 5 out of 11 or 5/11 C. 4 out of 11 or 4/11 9. What is the probability of choosing a blue paper if there are 3 red papers and 4 blue papers in the box? A. 4 out of 7 or 4/7 B. 3 out of 7 or 3/7 C. 0 out of 7 or zero probability 10. In the numbers 2,2,2,5,5,2,3 What is the probability of getting 2? A. 2/7 B. 3/7 C. 4/7 list out the ideas that help in maintaining social hormony in the community After an accident, Lavinia has been experiencing some aches and pains throughout her body. Her doctor says that she is healthy, but she still feels uncomfortable. Which of the following might help Lavinia reduce her pain? Why did white settlers not recognize Plains Indian claims to the land? In the context of supply chain management (scm) technologies, _____ enables business partners to send and receive information on business transactions I want to know about the history of The Sino-Japanese X Corp. and Y Corp. are separate entities. In Year 4, X Corp. extended a hostile tender offer directly to the shareholders of Y Corp. Y Corp. immediately issued bonds and offered to repurchase its own stock from its current shareholders at an amount higher than the current market price. The action by Y Corp. is best described as a(n) Issuance of stock strategy. Legal action strategy. Self-tender strategy. Compulsory share exchange strategy. Development refers to the pattern of continuity and change in human capabilities that occurs _____.